What is the equation, in point-slope form, of the line that
is perpendicular to the given line and passes through the
point (-4,-3)?

What Is The Equation, In Point-slope Form, Of The Line Thatis Perpendicular To The Given Line And Passes

Answers

Answer 1

Answer:

Step-by-step explanation:

We first need to find the slope of the line that is graphed. We can wither use the slope formula or you can use the slope triangle. From the upper point on the line (-1, 1) count down til you're on the same horizontal as the lower point on the line (0, -3). You have to count down 4 (which is -4) and over to the right 1 (which is +1). So -4/+1 = -4 and the slope is -4. That means that the perpendicular slope, the opposite reciprocal of that, is 1/4. Using that slope and the point (-4, -3), the point-slope form of the line is

[tex]y-(-3)=\frac{1}{4}(x-(-4))[/tex] which we can simplify a bit to

[tex]y+3=\frac{1}{4}(x+4)[/tex]. That's the line in point-slope form.


Related Questions

Evaluate the integral.

∫ 17/ (x^3 − 27) dx

Answers

Answer:

??????????????????????????

What is the volume of the solid figure?

help pls

Answers

Answer:

B, 4x6x7 and 1x3x3.

Step-by-step explanation:

First, remember the formula: V=LWH (Volume equals length times width times height). Lastly, find the equations that match that formula, therefor B, 4x6x7 and 1x3x3 matches the formula, so B  is your correct answer.

some 1 help me plz its algebra 1

Answers

5

Rise/run. When x is 0, y is 4. When x is 2, y is 14. Y changed by 10 and x changed by 2. 10/2=5

Complete each congruency statement and name the rule used. If you cannot show the triangles are congruent from the given information, leave the triangle's name blank and write CNBD for "Cannot be determined" in place of the rule. ∆ARE ≅ ∆_____ by _____

Answers

Answer:

add a picture so i can see .

Step-by-step explanation:

SOMONE HELP WITH MATH

Answers

Answer:

x = 28

Step-by-step explanation:

HFG = EFI

6x - 4 = 164

6x = 164 + 4

6x = 168

x = 168/6

x = 28

HELP ME PLSSS!
The graph below have a same shape what is the equation of the red graph?

A) g(x)=(3-x)2
B) g(x)=3-x2
C) g(x)=2-x2
D) g(x)=(2-x)2

Answers

Answer:  choice C)  g(x) = 2-x^2

============================================================

Explanation:

The y intercept of the blue curve f(x) is (0,5)

The y intercept of the red curve g(x) is (0,2)

We see that the y intercept has been shifted down 3 units when going from f(x) to g(x). Overall, the entire f(x) curve has been shifted down 3 units to get the g(x) curve.

Based on that, we would say

g(x) = f(x) - 3

g(x) = (5-x^2) - 3

g(x) = -x^2 + (5 - 3)

g(x) = -x^2 + 2

g(x) = 2 - x^2 ..... answer is choice C

Answer:

2-x^2

Step-by-step explanation:

pls answer i’ll give brainliest if it lets me

Answers

Answer:

32.25

Step-by-step explanation:

1 quart = 32 fluid ounces

12 months = 1 year

86 quarts/1 month = 1032 quarts/12 months

1032/32 = 32.25 fluid ounces

a solid wooden cube has 4.35cm long.calculate the volume of the cube​

Answers

Answer:

82.31

Step-by-step explanation:

I believe this is correct, if it isn't feel free to let me know and I will fix it. I'm sorry in advance if this is incorrect.

Will mark Brainlest hellpppp​

Answers

[tex]h(-3) - h( - 2) = - \frac{5}{ 8} \\ \\ [/tex]

Step-by-step explanation:

[tex]\boxed{h(x) = \frac{2 {x}^{2} - x + 1}{3x - 2}}[/tex]

[tex]h(2) = \frac{2 {(2)}^{2} - (2) + 1}{3(2) - 2} [/tex]

[tex]h(2) = \frac{2 {(4)} - 2 + 1}{6 - 2} [/tex]

[tex]h(2) = \frac{ 8 - 1}{4} [/tex]

[tex]h(2) = \frac{7}{4}[/tex]

[tex]h( - 3) = \frac{2 {( - 3)}^{2} - ( - 3) + 1}{3( - 3) - 2} \\ h( - 3) = \frac{2 (9) + 3 + 1}{ - 9- 2} \\ h( - 3) = \frac{18 + 4}{ - 11} \\ h( - 3) = \frac{22}{ - 11} \\ h(-3) = -2[/tex]

[tex]h( - 2) = \frac{2 {( - 2)}^{2} - ( - 2) + 1}{3( - 2) - 2} \\ h( - 2) = \frac{2 (4) + 2+ 1}{- 6 - 2} \\ h( - 2) = \frac{8 + 3}{- 8} \\ h( - 2) = \frac{11}{- 8} [/tex]

[tex]h(3) - h( - 2) = -2 - \frac{11}{- 8} \\ h(3) - h( - 2) =-2 + \frac{11}{ 8} \\ h(3) - h( - 2) = \frac{-2}{1}+ \frac{11}{ 8} \\ h(3) - h( - 2) = \frac{-16}{8} + \frac{11}{ 8} \\ h(3) - h( - 2) = \frac{-16+11}{8} \\ h(3) - h( - 2) = \frac{-5}{8} \\ - \frac{5}{ 8} [/tex]

Anita had $400 in her savings account when she went to college. Her parents will add $200 to her account each month.

Miguel had $25 in his savings account. His parents will double the amount in his account each month.

If Anita and Miguel do not take any money from their accounts, whose account will grow faster? Explain why.

Answers

Answer:

Miguel's account

Step-by-step explanation:

Miguel's account savings are doubled every month. Hence, they will eventually surpass the savings of Anita, even though Anita's account has more money atm.

Answer:

Miguel's account

Step-by-step explanation:

Even though Anita had more money at first compared to Miguel, Miguel savings will double each month while Anita will get only $200 each month. As a result, Miguel's account will grow faster compared to Anita's.

HELP !!!! look at the picture

Answers

Answer:

Option C:

g(x) = -3^x

Step-by-step explanation:

Here we can see the graphs of f(x) and the graph of g(x).

By only looking at that, we can see that g(x) is a reflection across the x-axis of f(x).

Remember that for a general function f(x), the reflection across the x-axis is written as:

g(x) = -f(x).

Then, if f(x) = 3^x

and g(x) = -f(x)

replacing f(x) we get:

g(x) = -(3^x) = -3^x

The correct option is C.

A set of data items is normally distributed with a mean of 400 and a standard deviation of 60. Find the data item in this distribution that corresponds to the following z-score:
z=3

Answers

Answer:

The data item is [tex]X = 580[/tex]

Step-by-step explanation:

Normal Probability Distribution:

Problems of normal distributions can be solved using the z-score formula.

In a set with mean [tex]\mu[/tex] and standard deviation [tex]\sigma[/tex], the z-score of a measure X is given by:

[tex]Z = \frac{X - \mu}{\sigma}[/tex]

The Z-score measures how many standard deviations the measure is from the mean. After finding the Z-score, we look at the z-score table and find the p-value associated with this z-score. This p-value is the probability that the value of the measure is smaller than X, that is, the percentile of X. Subtracting 1 by the p-value, we get the probability that the value of the measure is greater than X.

Mean of 400 and a standard deviation of 60.

This means that [tex]\mu = 400, \sigma = 60[/tex]

z=3

We have to find X when Z = 3. So

[tex]Z = \frac{X - \mu}{\sigma}[/tex]

[tex]3 = \frac{X - 400}{60}[/tex]

[tex]X - 400 = 3*60[/tex]

[tex]X = 580[/tex]

The data item is [tex]X = 580[/tex]

Find the value of x if x/3-2=6

Answers

Step-by-step explanation:

hope it is helpful to you

stay safe happy and healthy

[tex]\huge\textsf{Hey there!}[/tex]

[tex]\mathsf{\dfrac{x}{3}-2=6}[/tex]

[tex]\mathsf{= \dfrac{1}{3}x - 2 = 6}[/tex]

[tex]\textsf{ADD 2 BOTH SIDES}[/tex]

[tex]\mathsf{\dfrac{1}{3}x - 2 + 2 = 6 + 2}[/tex]

[tex]\textsf{CANCEL out: -2 + 2 because that gives you 1}[/tex]

[tex]\textsf{KEEP: 6 + 2 because that helps solve for x}[/tex]

[tex]\mathsf{6 + 2 = \boxed{\bf 8}}[/tex]

[tex]\mathsf{NEW \ EQUATION: \dfrac{1}{3}x = 8}[/tex]

[tex]\textsf{MULTIPLY 8 to BOTH SIDES}[/tex]

[tex]\mathsf{3\times\dfrac{1}{3}x=3\times8}[/tex]

[tex]\mathsf{CANCEL\ out: \ 3 \times \dfrac{1}{3} \ because \ that \ gives \ you \ 1}[/tex]

[tex]\mathsf{KEEP: \ 3 \times 8 \ because \ that\ gives \ you \ the \ value \ of \ x}[/tex]

[tex]\mathsf{\boxed{\bf x}= 3\times8}[/tex]

[tex]\mathsf{3\times8 = \boxed{\bf x}}[/tex]

[tex]\boxed{\boxed{\large\textsf{Answer: \huge \bf x = 24}}}\huge\checkmark[/tex]

[tex]\large\textsf{Good luck on your assignment and enjoy your day!}[/tex]

~[tex]\frak{Amphitrite1040:)}[/tex]

Help help help help help

Answers

Answer:

m(∠y) = 64°

Step-by-step explanation:

From the figure attached,

m(∠e) = 90°

m(∠b) + 67° = 180° [Linear pair of angles]

m(∠b) = 180 - 67

          = 113°

m(∠c) + 75° = 180°  [Linear pair of angles]

m(∠c) = 105°

m(∠a) = m(∠d)

By the property of a polygon,

Sum of the interior angles of a polygon is given by,

Sum of interior angles = (n - 2) × 180°

Here, n = number of sides of the polygon

For n = 5,

Sum of interior angles = (5 - 2)×180°

                                     = 540°

m(∠a) + m(∠b) + m(∠c) + m(∠d) + m(e) = 540°

2m(∠d) + 113° + 105° + 90° = 540°

2m(∠d) + 308 = 540°

2m(∠d) = 540 - 308

m(∠d) = 116°

m(∠d) + m(∠y) = 180°

m(∠y) + 116° = 180° [Linear pair of angles]

m(∠y) = 64°

10
17 A sequence starts at 300 and 40 is subtracted each timee.
300
260
2200
1800...
The sequence continues in the same way,
What is the first number in the sequence which is less than zerol?
[11]

Answers

9514 1404 393

Answer:

  -20

Step-by-step explanation:

300/40 = 7.5, so the 8th term will be the last positive term. It will be 300 -7×40 = 20. The 9th term will be 20 -40 = -20.

__

The sequence starts ...

  300, 260, 220, 180, 140, 100, 60, 20, -20, ...

Pencils cost x cents and erasers cost y cents each. Mary buys 7 pencils and 3 erasers. Write down the total cost in cents, in terms of x and y.

Answers

Answer:

7x + 3y.

Step-by-step explanation:

7 pencils cost 7x cents and 3 erasures cost 3y cents.

At : a.m. the angle of elevation of the sun for one city is . If the height of a monument is approximately , what is the length of the shadow it will cast at that time? Round to the nearest foot.

Answers

This question is incomplete, the complete question;

At 11:30 a.m. the angle of elevation of the sun for one city is 55.7°. If the height of a monument is approximately 555 ft, what is the length of the shadow it will cast at that time? Round to the nearest foot.

Answer:

the length of the shadow will be 379 ft

Step-by-step explanation:

Given the data in the question and as represented in the diagram below;

height of monument = 555 ft

angle of elevation = 55.7°

From the image below, this makes a right angled triangle

we know that the some of the interior angles of a triangle is 180

so

∠ABC + ∠BCA + ∠CAB = 180°

90° + 55.7° + ∠CAB  = 180°

∠CAB = 180° - 145.7°

∠CAB = 34.3°

Now, using sine rule;

BC / sinA = AB / sinC

so we substitute

BC / sin( 34.3°) = 555 / sin( 55.7° )

BC / 0.563526 = 555 / 0.826098

we cross multiply

BC × 0.826098 = 0.563526 × 555

BC × 0.826098 = 312.75693

BC = 312.75693 / 0.826098

BC = 378.595 ≈ 379 ft

Therefore, the length of the shadow will be 379 ft

Stephen has some money in a box ,if 3/8 of the money is #4.80,how much does he have in the box?​

Answers

If 3/8 is #4.80 then 1/8 would be #1.60 and 8/8 is #12.80

Which number line represents the solution set for the inequality? HELP PLEASE

Answers

Answer:

sorry i dont know next time ill surely help when i know.

Write an explicit formula for an, the nth term of the sequence 63,21,7

Answers

Answer:

63/3=2121/3=77/3=7 -

3

One hundred forty-eight ten thousandth is

Answers

0.0148 hope this helped
one hundred forty-eight ten thousandths is 0.0148

What is the length of each leg of the triangle below?
459
22
90°
45
O A. 11.12
B. 1
C. 15
D. 11
ET
F. 22

Answers

Answer:

option A

Step-by-step explanation:

since the given triangle is an isosceles triangle it's two remaining sides are equal

let the length of missing side be x

using pythagoras theorem

a^2 + b^2 = c^2

x^2 + x^2 = 22^2

2x^2 = 484

x^2 = 484/2

x = [tex]\sqrt{242}[/tex]

x = [tex]11\sqrt{2}[/tex]

Which expression is equivalent to xy^2/9?

Answers

Answer:

[tex]xy^\frac{2}{9} = x*\sqrt[9]{y^2}[/tex]

Step-by-step explanation:

Given

[tex]xy^\frac{2}{9}[/tex]

Required

The equivalent expression (see attachment)

We have:

[tex]xy^\frac{2}{9}[/tex]

Split

[tex]xy^\frac{2}{9} = x*y^\frac{2}{9}[/tex]

Apply the following laws of indices

[tex]y^\frac{m}{n} = \sqrt[n]{y^m}[/tex]

So, we have:

[tex]xy^\frac{2}{9} = x*\sqrt[9]{y^2}[/tex]

Hence (d) is correct

Answer:

Its D.

Step-by-step explanation:

Just took the quiz on EDGE2022


2. Tricky Flips sells a coin that promises to land on heads 3 out of every 4 times. If the coin is
flipped 20 times, which of the following is the number of times you should expect it to land
on head

Answers

Answer:

15

Step-by-step explanation:

Because 3/4 of 20 or 75% of 20 is 15

HELP ASAP PLS 10 points AND BRAINLEST

Answers

Answer:

ABC is congruent to EDC because m<3 = m<4 and m<1 = m<5

Step-by-step explanation:

m<3 and m<4 are vertical angles so they are congruent. m<1 and m<5 are alternate interior angles so they are also congruent.

Solve for X in the triangle. Round your answer to the nearest TENTH. (LISTING BRAINLIST PLZ HELP)​

Answers

Answer:

2.3 =x

Step-by-step explanation:

We know the opposite and adjacent sides.

Since this is a right triangle, we can use trig functions

tan 38 = opp/ adj

tan 38 = x/3

3 tan 38 = x

2.34385688= x

To the nearest tenth

2.3 =x

Answer:

x ≈ 3.9

General Formulas and Concepts:

Pre-Algebra

Order of Operations: BPEMDAS

Brackets Parenthesis Exponents Multiplication Division Addition Subtraction Left to Right  

Equality Properties

Multiplication Property of Equality Division Property of Equality Addition Property of Equality Subtraction Property of Equality

Trigonometry

[Right Triangles Only] SOHCAHTOA  [Right Triangles Only] tanθ = opposite over adjacent

Step-by-step explanation:

Step 1: Define

Identify variables

Angle θ = 38°

Opposite Leg = x

Adjacent Leg = 5

Step 2: Solve for x

Substitute in variables [tangent]:                                                                     tan38° = x/5[Multiplication Property of Equality] Multiply 5 on both sides:                      5tan38° = xRewrite:                                                                                                              x = 5tan38°Evaluate:                                                                                                           x = 3.90643Round:                                                                                                               x ≈ 3.9

What is an
equation of the line that passes through the points (0, -1) and (6, 1)?

Answers

Answer:

y = 1/3x -1

Step-by-step explanation:

The slope is  y2 - y1 / x2 - x1

y2 is 1

y1 is -1

x2 is 6

x1 is 0

So it is 1 - (-1) / 6 - 0, or 2/6, or 1/3

The slope is 1/3.

The y-intercept is the place where x is 0. It shows the y-intercept here as

(0, -1),  The y-intercept is  -1

Slope form is y = mx + b. m is the slope, 1/3, and b is the y-intercept, -1

So the equation is: y = 1/3x -1

approximate 10.54 to the nearest ten​

Answers

Answer:

11 because <.5 is rounded to the next ten

A) work out the value of g.
Give your answer in standard form correct to three significant figures.
B) work out the new value of g. Give your answer in standard form correct to 3 significant figures. (M is increased by 8% and T is increased by 5%).

Answers

Answer:

4547.14

Step-by-step explanation:

m increased by %8 so it'll be

[tex]6.588 \times {10}^{ - 5} [/tex]

and t will be

[tex]1.785 \times {10}^{ - 6} [/tex]

so G =

[tex] \sqrt{ \frac{(6.588 \times {10}^{ - 5}) }{ {(1.785 \times {10}^{ - 6}) }^{2} } } [/tex]

G= 4547

At Marco’s school, 5/8 of the students are in the band. What percent of the students are in the band?

Answers

63 percent of students are in the band!
Other Questions
Which statement accurately describes the Virginia Plan?A. There would be two houses in Congress, and representatives would be assigned based on state population. Which class is better, math or science (in your opinion)comment/answer your fav i need this for a survey Express 60g as a percentage of 3kg? Select the phrase in the excerpt that best supports this inference: womens right to vote Which sentence is an example of an objective summary?A. Mortimer is a witty writer who adds humor to history.B. It must have been horrible to live in Elizabethan times.C. Quality health care today is wonderful, but overpriced.D. Modern medicine has changed dramatically since the 1500s. The length and width of a rectangular boxare represented by 2x+3 and 3x + 1 respectively.What is the area of the box expressed as a polynomialin standard form? Someone help me with this question please Line segment EG is partitioned by point F in the ratio 1:3. Point E is at E (0, 4), and point F is at (1, 3). What are the coordinates of point G? (1, 5) (2, 2) (3, 1) (4, 0) Solve the expression one half x (48 6) 2 + 5 using PEMDAS. 7 8 9 10 Jada asked some students at her school how many hours they spent watching television last week, to the nearest hour. Here are a box plot and a histogram for the data she collected.If anyone trolls imma be very sad :( if a line with an angle of inclination 120 and passes trough (3,1),then what is the equation of a line The technique for linking a manufacturer's operations with those of all its strategic suppliers and its key intermediaries and customers to enhance efficiency and effectiveness is If abushel of apples weight from 48 to 54 pounds and a bushel of melons weight from 80 to 90 pounds, what is the smallest ratio between the weight of a bushel of apples and a bushel of melon? IMPORTANCE OF ENZYMES?? Which of the following statements is FALSE? A. Time management and planning is a crucial component of the research paper process. B. Research cybercards are an online method of taking notes and tracking sources. C. Research note cards should be reserved for facts and data information only. D. A weekly planner or calendar is an excellent timeline device for organizing tasks and due dates. Please select the best answer from the choices provided A B C D 1, My house is bigger than your house. -> Your house is................................................................................................................2, The black is cheaper than the red car.-> The red car..........................................................................................................................3, There is a sink , a tub and a shower in the bathroom.-> The bathroom............................................................................................................4, No one in the group is taller than Trung.-> Trung.................................................................................................................................5, Do you have a better refrigerator than this?-> Is this............................................................................................................................... Find length bc. Give your answer to 1 decimal place 11cm 64 trigonometry Based on the graph, which of these conclusions is correct? (2 points)aHeat is not added in portions BC and DE. bHeat is not added in portions AB and CD. cChange of state takes place in portions BC and DE. dChange of state takes place in portions AB and CD. log (3x+5)=log (2x-7) What is the name given to software that decodes information from a digital file so that a media player can display the file? hard drive plug-in flash player MP3